+1 Daumen
268 Aufrufe

Aufgabe:

Ich soll das asymptotische Wachstum in Theta Notation angeben für die Laufzeit.

function A(n,a):
if n = 0 then
 return a
fi
x := A(n /2,1) + A(n/2,2) + A(n/2,3)
for i = 1 to n do
  x := x + 1
od
return x

Meine Überlegung:

1. Idee:
mastertheorem
a= 3 b=2 c=1
3 > 2^1=2
somit t(n) \in Theta ( n^(log3/log2) = n^(log_2 3)

Stimmt das?

Avatar von

1 Antwort

0 Daumen

Antwort mit KI erzeugt

Analyse der Laufzeit eines Algorithmus

Um die Laufzeit des gegebenen Algorithmus zu analysieren und das asymptotische Wachstum in Theta-Notation anzugeben, betrachten wir zunächst den gegebenen Code und deine Überlegungen.

Die rekursive Funktion sieht wie folgt aus:

plaintext
function A(n,a):
if n = 0 then
  return a
fi
x := A(n /2,1) + A(n/2,2) + A(n/2,3)
for i = 1 to n do
  x := x + 1
od
return x


Bei der Analyse des Codes ist folgendes zu beachten:

- Die Rekursion ruft sich selbst dreimal auf, jedes Mal mit dem ersten Argument \(n / 2\). Dies gibt an, dass die Größe des Problems bei jedem Rekursionsschritt halbiert wird.
- Nachdem alle Rekursionsaufrufe abgeschlossen sind, führt der Algorithmus eine Schleife aus, die von 1 bis \(n\) läuft, und erhöht \(x\) bei jedem Durchlauf um 1. Diese Schleife stellt eine lineare Zeitkomplexität \(\Theta(n)\) dar.

Um das asymptotische Wachstum der Funktion zu bestimmen, verwenden wir deine Idee, das Mastertheorem anzuwenden.

Laut dem Mastertheorem, wenn wir eine Rekurrenz der Form

\(T(n) = aT\left(\frac{n}{b}\right) + f(n)\)

haben, wobei:
- \(a \geq 1\) die Anzahl der Unterprobleme in der Rekursion,
- \(b > 1\) der Faktor ist, um den die Problemgröße bei jedem Rekursionsschritt reduziert wird,
- \(f(n)\) die Kosten außerhalb der Rekursion,

dann können wir die Laufzeit \(T(n)\) unter bestimmten Bedingungen in der Theta-Notation angeben.

Deine Angaben:
- \(a = 3\) (da der Algorithmus dreimal rekursiv aufgerufen wird),
- \(b = 2\) (da das Problem bei jedem Rekursionsschritt halbiert wird),
- \(c = 1\) (als Exponent in \(n^c\), der die Kosten \(f(n)\) darstellt, das entspricht aber nicht dem Standardform sondern es ist \(f(n) = \Theta(n)\) wegen der Schleife).

Nach der dritten Regel des Mastertheorems, falls \(f(n) = \Omega(n^c)\) für ein \(c > \log_b a\), dann ist \(T(n) = \Theta(f(n))\). In deinem Fall ist \(\log_2 3\) tatsächlich der Exponent, gegen den wir \(n\) vergleichen sollten, dann vergleichen wir \(f(n) = \Theta(n)\) mit \(n^{\log_2 3}\).

Allerdings entspricht der lineare Teil \(\Theta(n)\) nicht direkt \(n^{\log_2 3}\), da er \(f(n)\) repräsentiert. Der wesentliche Punkt hier ist, dass \(f(n)\) in diesem Fall aus der linearen Schleife \(n\) und nicht aus der potentiellen Kostenfunktion \(n^c\) kommt.

Du hast richtig erkannt, dass wegen der drei rekursiven Aufrufe und der Tatsache, dass das Problem bei jedem Schritt halbiert wird, die Kostenstruktur einem Fall des Mastertheorems entsprechen könnte. Jedoch, deine Interpretation der Rolle vom \(f(n)\) bzw. von \(c\) war nicht korrekt identifiziert in Bezug auf die Bedingung des Theorems.

Die korrekte Analyse zeigt, dass
- Da \(a = 3\), \(b = 2\), und die außerhalb der Rekursion liegenden Kosten \(f(n) = \Theta(n)\) betragen, befinden wir uns im dritten Fall des Mastertheorems. \(f(n) = \Omega(n^c)\), wo \(c = 1 > \log_2 3\), scheint nicht direkt hier anwendbar, weil \(f(n)\) tatsächlich \(\Theta(n)\) ist und nicht durch \(n^{\log_2 3}\) übertroffen wird.

Allerdings macht hier ein Detail einen Unterschied. Wegen des linearen Schleifenkörpers haben wir:
- \(T(n) = 3T(\frac{n}{2}) + \Theta(n)\).

Das bedeutet, dass die Rekursion dominiert, da:
- \(3 > 2^1\),

und das Gesamtwachstum wird:
- \(T(n) \in \Theta(n^{\log_2 3})\).

Daher war deine Schlussfolgerung in Bezug auf das Wachstum von \(T(n)\) als \(n^{\log_2 3}\) korrekt, aber die Erläuterung, wie es zu diesem Ergebnis kommt, benötigte eine Klärung. Die Kosten \(f(n) = \Theta(n)\) bestimmen nicht direkt das asymptotische Wachstum in diesem Fall, sondern die Anzahl und Größe der Rekursionsaufrufe im Vergleich zu den Kosten der Schleife außerhalb der Rekursion führen uns zum \(\Theta(n^{\log_2 3})\).
Avatar von 2,9 k

Ein anderes Problem?

Stell deine Frage

Willkommen bei der Stacklounge! Stell deine Frage einfach und kostenlos

x
Made by a lovely community